LSAT PREP TEST QUESTIONS Flashcards
(658 cards)
LSAT Logical Reasoning Prep Test 52 Question #1 Certain companies require their managers to rank
workers in the groups they supervise from best to
worst, giving each worker a unique ranking based on
job performance. The top 10 percent of the workers
in each group are rewarded and the bottom 10
percent are penalized or fired. But this system is
unfair to workers. Good workers could receive low
rankings merely because they belong to groups of
exceptionally good workers. Furthermore, managers
often give the highest rankings to workers who share
the manager’s interests outside of work.
Which one of the following most accurately expresses
the conclusion drawn in the argument?
(A) Some companies require their managers to give
unique rankings to the workers they supervise.
(B) Under the ranking system, the top 10 percent of
the workers in each group are rewarded and the
bottom 10 percent are penalized or fired.
(C) The ranking system is not a fair way to determine
penalties or rewards for workers.
(D) Workers in exceptionally strong work groups are
unfairly penalized under the ranking system.
(E) Managers often give the highest rankings to
workers who share the manager’s outside
interests.
Correct Answer: C
C Main Point
This argument disagrees with the compensation system companies use requiring managers to first rank their workers from best to worst, and then reward the top 10 percent in each group and penalize or fire the workers in the bottom 10 percent. The argument concludes that this system is unfair to workers. The premises are that the rankings depend too much on the quality of the workers with whom each worker is grouped. Also, managers often rank workers for the wrong reasons, such as affinity.
A. No. This is a premise.
B. No. This is a premise.
C. Yes. This is a good restatement of the disagreement.
D. No. This is a premise.
E. No. This is a premise.
LSAT Logical Reasoning Prep Test 52 Question #2 Psychologist: A study of 436 university students
found that those who took short naps
throughout the day suffered from insomnia
more frequently than those who did not.
Moreover, people who work on commercial
fishing vessels often have irregular sleep
patterns that include frequent napping, and
they also suffer from insomnia. So it is very
likely that napping tends to cause insomnia.
The reasoning in the psychologist’s argument is most
vulnerable to criticism on the grounds that the
argument
(A) presumes, without providing justification, that
university students suffer from insomnia
more frequently than do members of the
general population
(B) presumes that all instances of insomnia have
the same cause
(C) fails to provide a scientifically respectable
definition for the term “napping”
(D) fails to consider the possibility that frequent
daytime napping is an effect rather than a
cause of insomnia
(E) presumes, without providing justification, that
there is such a thing as a regular sleep pattern
for someone working on a commercial fishing
vessel
Correct Answer: D
D Flaw
The argument presents a causal interpretation of evidence. The psychologist concludes that napping is likely to cause insomnia because two groups of people who nap more suffer more from insomnia.
A. No. The argument does not compare university students to the general population, but rather university students who nap to those who do not.
B. No. The argument does not assume that napping is the only cause of insomnia, just that it tends to cause insomnia.
C. No. The argument uses the term “napping” consistently and does not need to define it.
D. Yes. The argument fails to rule out the possibility that the causal direction might be reversed.
E. No. This is not essential to the argument.
LSAT Logical Reasoning Prep Test 52 Question #3 Whenever Joe’s car is vacuumed, the employees of
K & L Auto vacuum it; they are the only people who
ever vacuum Joe’s car. If the employees of K & L Auto
vacuumed Joe’s car, then Joe took his car to K & L
Auto to be fixed. Joe’s car was recently vacuumed.
Therefore, Joe took his car to K & L Auto to be fixed.
The pattern of reasoning exhibited by the argument
above is most similar to that exhibited by which one
of the following?
(A) Emily’s water glass is wet and it would be wet
only if she drank water from it this morning.
Since the only time she drinks water in the
morning is when she takes her medication,
Emily took her medication this morning.
(B) Lisa went to the hair salon today since either
she went to the hair salon today or she went
to the bank this morning, but Lisa did not go
to the bank this morning.
(C) There are no bills on John’s kitchen table. Since
John gets at least one bill per day and he
always puts his bills on his kitchen table,
someone else must have checked John’s mail
today.
(D) Linda is grumpy only if she does not have her
coffee in the morning, and Linda does not
have her coffee in the morning only if she
runs out of coffee. Therefore, Linda runs out
of coffee only on days that she is grumpy.
(E) Jeff had to choose either a grapefruit or cereal
for breakfast this morning. Given that Jeff is
allergic to grapefruit, Jeff must have had
cereal for breakfast this morning.
Correct Answer: A
A Parallel
Diagram the argument. Joe’s car is vacuumed → K & L vacuumed it. K & L vacuumed it → Joe took it to K & L to be fixed. The argument demonstrates that the sufficient factor (Joe’s car is vacuumed) is valid. Therefore, the result (Joe took his car to K & L to be fixed) must also be true.
A. Yes. Emily’s glass is wet → she drank water from it this morning → she took her medication. Emily’s glass is wet, so she must have taken her medication.
B. No. There is no either/or option in the original argument.
C. No. This answer choice does not match the structure of the original argument.
D. No. This answer choice demonstrates flawed reasoning and the original argument does not.
E. No. There is no either/or option in the original argument.
LSAT Logical Reasoning Prep Test 52 Question #4 Editorialist: In a large corporation, one of the
functions of the corporation’s president is to
promote the key interests of the shareholders.
Therefore, the president has a duty to keep the
corporation’s profits high.
Which one of the following, if true, would most
strengthen the editorialist’s argument?
(A) Shareholders sometimes will be satisfied even
if dividends paid to them from company
profits are not high.
(B) The president and the board of directors of a
corporation are jointly responsible for
advancing the key interests of the
shareholders.
(C) Keeping a corporation’s profits high is likely to
advance the important interests of the
corporation’s shareholders.
(D) In considering where to invest, most potential
shareholders are interested in more than just
the profitability of a corporation.
(E) The president of a corporation has many
functions besides advancing the important
interests of the corporation’s shareholders.
Correct Answer: C
C Strengthen
The editorialist concludes that the president has a duty to keep the corporation’s profits high. The premise is that one of the functions of the president is to promote the key interests of the stockholders. The argument would benefit from a connection between the key interests of the stockholders and high profits.
A. No. This answer choice would, if anything, weaken the argument.
B. No. The argument is concerned with the president and what he/she should do, not the board of directors.
C. Yes. This answer choice provides a strong connection between the key interests of the shareholders and keeping the profits high.
D. No. The argument does not imply that profitability is the only interest of the shareholders.
E. No. Like (D), the argument does not imply that advancing the important interests of shareholders is the president’s only responsibility.
LSAT Logical Reasoning Prep Test 52 Question #5 Everyone in Biba’s neighborhood is permitted to
swim at Barton Pool at some time during each day
that it is open. No children under the age of 6 are
permitted to swim at Barton Pool between noon and
5 P.M. From 5 P.M. until closing, Barton Pool is
reserved for adults only.
If all the sentences above are true, then which one of
the following must be true?
(A) Few children under the age of 6 live in Biba’s
neighborhood.
(B) If Biba’s next-door neighbor has a child under
the age of 6, then Barton Pool is open before
noon.
(C) If most children who swim in Barton Pool
swim in the afternoon, then the pool is
generally less crowded after 5 P.M.
(D) On days when Barton Pool is open, at least
some children swim there in the afternoon.
(E) Any child swimming in Barton Pool before
5 P.M. must be breaking Barton Pool rules.
Correct Answer: B
B Inference
This is a connect-the-facts inference with conditionals. Live in Biba’s neighborhood → permitted to swim at some time each day. Under 6 → can’t swim between noon and 5 P.M. Child → can’t swim from 5 P.M. until closing.
A. No. You can’t prove how many, if any, children live in Biba’s neighborhood.
B. Yes. Children under 6 can’t swim in the pool from noon until closing. Yet, as the passage says, everyone that lives in the neighborhood must be able to swim at some time during the day. This child would have to be able to swim before noon.
C. No. The argument does not provide any information to determine how crowded the pool will be.
D. No. The argument does not explain who lives in Biba’s neighborhood. It could be a neighborhood with no children.
E. No. The pool could be open before noon. Also, it’s only children under 6 that are not allowed to swim between noon and 5 P.M.
LSAT Logical Reasoning Prep Test 52 Question #6 Beck: Our computer program estimates municipal
automotive use based on weekly data. Some
staff question the accuracy of the program’s
estimates. But because the figures it provides
are remarkably consistent from week to week,
we can be confident of its accuracy.
The reasoning in Beck’s argument is flawed in that it
(A) fails to establish that consistency is a more
important consideration than accuracy
(B) fails to consider the program’s accuracy in
other tasks that it may perform
(C) takes for granted that the program’s output
would be consistent even if its estimates were
inaccurate
(D) regards accuracy as the sole criterion for
judging the program’s value
(E) fails to consider that the program could
produce consistent but inaccurate output
Correct Answer: E
E Flaw
The purpose of this argument is to disagree with a claim. Beck concludes that, despite what some of the staff says, they can be confident of their computer program’s accuracy because the figures that the program provides are consistent from week to week. Beck equates consistency with accuracy.
A. No. The argument doesn’t claim that consistency is more important than accuracy. It equates the two.
B. No. The argument is concerned with only one task: estimating the municipal automotive use.
C. No. The argument takes for granted that, because the output is consistent, the program must be accurate.
D. No. The argument is concerned with only the accuracy of the program. It doesn’t make any claims about the program’s value in general.
E. Yes. The argument assumes that, because the output is consistent, the program must be accurate.
LSAT Logical Reasoning Prep Test 52 Question #7 Inertia affects the flow of water pumped through a
closed system of pipes. When the pump is first
switched on, the water, which has mass, takes time to
reach full speed. When the pump is switched off,
inertia causes the decrease in the water flow to be
gradual. The effects of inductance in electrical
circuits are similar to the effects of inertia in water
pipes.
The information above provides the most support
for which one of the following?
(A) The rate at which electrical current flows is
affected by inductance.
(B) The flow of electrical current in a circuit
requires inertia.
(C) Inertia in the flow of water pumped by an
electrically powered pump is caused by
inductance in the pump’s circuits.
(D) Electrical engineers try to minimize the effects
of inductance in electrical circuits.
(E) When a water pump is switched off it
continues to pump water for a second or two
Correct Answer: A
A Inference
The argument describes how inertia affects the flow of water pumped through a closed system of pipes. It then equates the effects of inductance in electrical circuits with the effects of inertia in water pipes.
A. Yes. If the effects of inductance are similar to the effects of inertia, and inertia affects the flow of water, then inductance will affect the flow of electrical current.
B. No. Inertia refers to water and pipes, not the flow of electrical current.
C. No. You cannot prove how inductance affects inertia from the information provided.
D. No. You cannot prove anything about electrical engineers from the information provided.
E. No. This choice is too strong. All you can prove is that the inertia causes the decrease in the water flow to be gradual. You cannot prove how long this takes.
LSAT Logical Reasoning Prep Test 52 Question #8 Journalist: To reconcile the need for profits sufficient
to support new drug research with the moral
imperative to provide medicines to those who
most need them but cannot afford them, some
pharmaceutical companies feel justified in
selling a drug in rich nations at one price and
in poor nations at a much lower price. But this
practice is unjustified. A nation with a low
average income may still have a substantial
middle class better able to pay for new drugs
than are many of the poorer citizens of an
overall wealthier nation.
Which one of the following principles, if valid, most
helps to justify the journalist’s reasoning?
(A) People who are ill deserve more consideration
than do healthy people, regardless of their
relative socioeconomic positions.
(B) Wealthy institutions have an obligation to
expend at least some of their resources to
assist those incapable of assisting themselves.
(C) Whether one deserves special consideration
depends on one’s needs rather than on
characteristics of the society to which one
belongs.
(D) The people in wealthy nations should not have
better access to health care than do the people
in poorer nations.
(E) Unequal access to health care is more unfair
than an unequal distribution of wealth.
Correct Answer: C
C Principle Strengthen
The purpose of this argument is to disagree with a claim. The journalist thinks that the pharmaceutical companies are not justified in selling a drug in rich nations at one price and in poor nations at another price. The journalist’s premise is that many individuals in poorer nations might be better able to pay for new drugs than poorer individuals in nations with higher overall wealth.
A. No. The argument doesn’t compare ill people to healthy people.
B. No. This answer choice supports the position that the argument is designed to disagree with.
C. Yes. This answer choice points out that special consideration should be provided to individuals when individuals within a society might not have the same amount of resources as the society does, on average.
D. No. This answer choice is not relevant to the argument.
E. No. The argument does not discuss the fairness of unequal distribution of wealth.
LSAT Logical Reasoning Prep Test 52 Question #9 9. Robert: The school board is considering adopting a
year-round academic schedule that eliminates
the traditional three-month summer vacation.
This schedule should be adopted, since
teachers need to cover more new material
during the school year than they do now.
Samantha: The proposed schedule will not permit
teachers to cover more new material. Even
though the schedule eliminates summer
vacation, it adds six new two-week breaks, so
the total number of school days will be about
the same as before.
Which one of the following, if true, is a response
Robert could make that would counter Samantha’s
argument?
(A) Teachers would be willing to accept
elimination of the traditional three-month
summer vacation as long as the total vacation
time they are entitled to each year is not
reduced.
(B) Most parents who work outside the home find
it difficult to arrange adequate supervision
for their school-age children over the
traditional three-month summer vacation.
(C) In school districts that have adopted a yearround schedule that increases the number of
school days per year, students show a deeper
understanding and better retention of new
material.
(D) Teachers spend no more than a day of class
time reviewing old material when students
have been away from school for only a few
weeks, but have to spend up to a month of
class time reviewing after a three-month
summer vacation.
(E) Students prefer taking a long vacation from
school during the summer to taking more
frequent but shorter vacations spread
throughout the year.
Correct Answer: D
D Weaken
Robert is trying to solve a problem and Samantha claims that he hasn’t solved it. Robert concludes that the school board should adopt a year-round academic schedule because teachers need to cover more new material during the school year than they do now. Samantha claims that the new school schedule won’t permit the teachers to cover any more new material because the amount of vacation will be the same as before, just in a different configuration. The students will have six two-week breaks, instead of one three-month break.
A. No. This doesn’t address the problem of needing to cover more new material.
B. No. This answer choice does not address the problem of needing to cover more new material.
C. No. It’s nice that students show a deeper understanding of the material in year-round schools, but the answer choice does not address the issue at hand: needing to cover more new material.
D. Yes. If the teachers have to spend significantly less class time reviewing material after the short breaks in the year-round schedule than after the long break in the traditional schedule, then teachers will have more class days to cover new material.
E. No. Student preference is not relevant. The disagreement is about the amount of new material that could be covered.
LSAT Logical Reasoning Prep Test 52 Question #10 In order to reduce traffic congestion and raise
revenue for the city, the mayor plans to implement a
charge of $10 per day for driving in the downtown
area. Payment of this charge will be enforced using a
highly sophisticated system that employs digital
cameras and computerized automobile registration.
This system will not be ready until the end of next
year. Without this system, however, mass evasion of
the charge will result. Therefore, when the mayor’s
plan is first implemented, payment of the charge will
not be effectively enforced.
Which one of the following is an assumption on
which the argument depends for its conclusion to be
properly drawn?
(A) The mayor’s plan to charge for driving
downtown will be implemented before the
end of next year.
(B) The city will incur a budget deficit if it does
not receive the revenue it expects to raise
from the charge for driving downtown.
(C) The plan to charge for driving downtown
should be implemented as soon as payment of
the charge can be effectively enforced.
(D) Raising revenue is a more important
consideration for the city than is reducing
traffic congestion.
(E) A daily charge for driving downtown is the
most effective way to reduce traffic
congestion.
Correct Answer: A
A Necessary Assumption
The argument concludes that the mayor’s plan to reduce congestion and raise revenue by charging $10 per day for driving in the downtown area will not be effectively enforced when it is first implemented. The premises are that payment will be enforced by a highly sophisticated system that will not be ready until the end of next year, and that many people will avoid paying the charge without this system in place.
A. Yes. It is essential to the argument that the plan be in place before the system is ready at the end of next year.
B. No. The argument does not discuss the possibility of a budget deficit.
C. No. The argument is concerned with whether the plan will be effectively enforced when it is first implemented, not when it should be implemented.
D. No. The argument does not compare the importance of raising revenue versus reducing traffic congestion.
E. No. Too strong. The argument doesn’t need to say that a daily charge is the most effective way to reduce traffic congestion.
LSAT Logical Reasoning Prep Test 52 Question #11 A recent study revealed that the percentage of people
treated at large, urban hospitals who recover from
their illnesses is lower than the percentage for people
treated at smaller, rural hospitals.
Each of the following, if true, contributes to an
explanation of the difference in recovery rates
EXCEPT:
(A) Because there are fewer patients to feed,
nutritionists at small hospitals are better able
to tailor meals to the dietary needs of each
patient.
(B) The less friendly, more impersonal atmosphere
of large hospitals can be a source of stress for
patients at those hospitals.
(C) Although large hospitals tend to draw doctors
trained at the more prestigious schools, no
correlation has been found between the
prestige of a doctor’s school and patients’
recovery rate.
(D) Because space is relatively scarce in large
hospitals, doctors are encouraged to minimize
the length of time that patients are held for
observation following a medical procedure.
(E) Doctors at large hospitals tend to have a
greater number of patients and consequently
less time to explain to staff and to patients
how medications are to be administered.
Correct Answer: C
C Resolve/Explain
There is a discrepancy in recovery rates between people treated at large, urban hospitals and people treated at smaller, rural hospitals. People treated at smaller, rural hospitals have a higher rate of recovery. Eliminate any answer choices that help resolve the discrepancy.
A. No. If the patients at smaller hospitals are more likely to get fed according to their dietary needs, they might be more likely to recover from their illnesses.
B. No. If patients at larger hospitals are more stressed than patients at smaller hospitals, that might adversely affect their recovery rate.
C. Yes. This answer choice states that there has been no correlation found between the prestige of a doctor’s school and patients’ recovery rates. In no way is this relevant to explaining the differing recovery rates of patients treated at larger or smaller hospitals.
D. No. If patients are not observed for as long, there is a greater chance that an unobserved complication might occur, thus adversely affecting the recovery rate for patients treated at larger hospitals.
E. No. If the staff and patients do not get explanations about the administration of their medications, improper administration of these medications is more likely to occur. This would adversely affect the recovery rates of patients treated at larger hospitals.
LSAT Logical Reasoning Prep Test 52 Question #12 12. Perry: Worker-owned businesses require workers to
spend time on management decision-making
and investment strategy, tasks that are not
directly productive. Also, such businesses have
less extensive divisions of labor than do
investor-owned businesses. Such inefficiencies
can lead to low profitability, and thus increase
the risk for lenders. Therefore, lenders seeking
to reduce their risk should not make loans to
worker-owned businesses.
Which one of the following, if true, most seriously
weakens Perry’s argument?
(A) Businesses with the most extensive divisions of
labor sometimes fail to make the fullest use of
their most versatile employees’ potential.
(B) Lenders who specialize in high-risk loans are
the largest source of loans for worker-owned
businesses.
(C) Investor-owned businesses are more likely
than worker-owned businesses are to receive
start-up loans.
(D) Worker-owned businesses have traditionally
obtained loans from cooperative lending
institutions established by coalitions of
worker-owned businesses.
(E) In most worker-owned businesses, workers
compensate for inefficiencies by working
longer hours than do workers in investorowned businesses.
Correct Answer: E
E Weaken
Perry concludes that lenders who are seeking to reduce their risk should not make loans to worker-owned businesses. This is because worker-owned businesses require workers to spend time on management and investment, which are not directly productive. Worker-owned businesses also have less extensive divisions of labor than do investor-owned businesses. These inefficiencies can lead to low profitability, which would increase risk for lenders. An issue with Perry’s argument is that, just because inefficiencies can lead to low profitability, it doesn’t mean that they will. Maybe there is something unique about worker-owned businesses that will overcome this problem.
A. No. Businesses with the most extensive divisions of labor can sometimes fail to make the fullest use of their most versatile employees’ potential and still be more efficient, on average, than the worker-owned businesses and still be, on average, the safer investment.
B. No. This answer choice is a description of the lenders who do make loans to worker-owned businesses. It does not address the warning against lending to worker-owned businesses.
C. No. The argument is not concerned with start-up loans, nor is it concerned with who actually gets loans.
D. No. This answer choice does not give a reason as to why worker-owned businesses might be a less risky investment than the argument claims.
E. Yes. If the workers work longer hours, the inefficiencies might be compensated for, which means that the inefficiencies won’t necessarily lead to low profitability. If they don’t lead to low profitability, then the risk will not necessarily increase for lenders.
LSAT Logical Reasoning Prep Test 52 Question #13 13. Some paleontologists believe that certain species of
dinosaurs guarded their young in protective nests
long after the young hatched. As evidence, they cite
the discovery of fossilized hadrosaur babies and
adolescents in carefully designed nests. But similar
nests for hatchlings and adolescents are constructed
by modern crocodiles, even though crocodiles guard
their young only for a very brief time after they
hatch. Hence, .
Which one of the following most logically completes
the argument?
(A) paleontologists who believe that hadrosaurs
guarded their young long after the young
hatched have no evidence to support this
belief
(B) we will never be able to know the extent to
which hadrosaurs guarded their young
(C) hadrosaurs guarded their young for at most
very brief periods after hatching
(D) it is unclear whether what we learn about
hadrosaurs from their fossilized remains tells
us anything about other dinosaurs
(E) the construction of nests for hatchlings and
adolescents is not strong evidence for the
paleontologists’ belief
Correct Answer: E
E Main Point
This argument is designed to disagree with a claim. The argument disagrees with some paleontologists, who believe that certain dinosaurs guarded their young in protective nests long after the young hatched. The evidence cited for the paleontologists’ claim is the discovery of fossilized hadrosaur babies and adolescents in carefully designed nests. In disagreeing with these paleontologists, the argument notes that modern crocodiles construct similar nests, even though these crocodiles don’t guard their young for long. The argument seems to be leading to the conclusion that the evidence of fossilized nests is not enough to claim that the dinosaurs guarded their young long after the young hatched.
A. No. This answer choice is too strong. The paleontologists do have some evidence, just not enough to fully support their conclusion.
B. No. This answer choice is too strong. The evidence cited is not strong enough for the paleontologists’ conclusion. That’s not to say that we will never know the extent to which hadrosaurs guarded their young.
C. No. This answer choice is too strong. There is not enough evidence to know that they guarded their young for large periods of time. That’s not to say that hadrosaurs couldn’t have actually guarded their young for large periods of time.
D. No. This is not the disagreement. The paleontologists are making a claim about a certain species of dinosaurs, not all dinosaurs.
E. Yes. The argument disagrees with the paleontologists as to the strength of the evidence that supports their belief.
LSAT Logical Reasoning Prep Test 52 Question #14 14. For one academic year all the students at a high
school were observed. The aim was to test the
hypothesis that studying more increased a student’s
chances of earning a higher grade. It turned out that
the students who spent the most time studying did
not earn grades as high as did many students who
studied less. Nonetheless, the researchers concluded
that the results of the observation supported the
initial hypothesis.
Which one of the following, if true, most helps to
explain why the researchers drew the conclusion
described above?
(A) The students who spent the most time
studying earned higher grades than did some
students who studied for less time than the
average.
(B) The students tended to get slightly lower
grades as the academic year progressed.
(C) In each course, the more a student studied, the
better his or her grade was in that course.
(D) The students who spent the least time studying
tended to be students with no more than
average involvement in extracurricular
activities.
(E) Students who spent more time studying
understood the course material better than
other students did.
Correct Answer: C
C Resolve/Explain
The apparent paradox is that when researchers tested the hypothesis that studying more increased a student’s chances of earning a higher grade, the students who spent the most time studying did not earn grades as high as did many students who studied less. Yet, the researchers concluded that the results supported their hypothesis.
A. No. This doesn’t resolve the apparent paradox. The passage says that many students who studied less earned higher grades. This still leaves room for some students who studied less to get lower grades, while still maintaining the seeming paradox.
B. No. If all the students tended to get slightly lower grades as the year progressed, the problem still remains that many students who studied less will have higher grades than the students who spent the most time studying.
C. Yes. This resolves the apparent paradox. If each individual student does better in a given class if he or she studies, then it looks like studying more will increase a student’s change of getting a higher grade, even if the students who study the most get lower grades than many who don’t study as much. The hypothesis was about an individual’s grades, while the statistics were about the students considered in groups.
D. No. This does not resolve the apparent paradox.
E. No. This does not resolve the apparent paradox as well as (C) because it doesn’t directly connect studying with an individual’s chances of getting a better grade in a given class.
LSAT Logical Reasoning Prep Test 52 Question #15 15. Researchers had three groups of professional cyclists
cycle for one hour at different levels of intensity.
Members of groups A, B, and C cycled at rates that
sustained, for an hour, pulses of about 60 percent, 70
percent, and 85 percent, respectively, of the
recommended maximum pulse rate for recreational
cyclists. Most members of Group A reported being
less depressed and angry afterward. Most members of
Group B did not report these benefits. Most
members of Group C reported feeling worse in these
respects than before the exercise.
Which one of the following is most strongly
supported by the information above?
(A) The higher the pulse rate attained in sustained
exercise, the less psychological benefit the
exercise tends to produce.
(B) The effect that a period of cycling has on the
mood of professional cyclists tends to depend
at least in part on how intense the cycling is.
(C) For professional cyclists, the best exercise from
the point of view of improving mood is
cycling that pushes the pulse no higher than
60 percent of the maximum pulse rate.
(D) Physical factors, including pulse rate,
contribute as much to depression as do
psychological factors.
(E) Moderate cycling tends to benefit professional
cyclists physically as much or more than
intense cycling
Correct Answer: B
B Inference
Find the answer choice supported by the passage.
A. No. This answer choice is too strong. The argument talks only about cycling, not about exercise in general. In addition, you can’t prove that in every instance the higher the pulse rate, the less psychological benefit produced. What if having a pulse rate of 60 percent of the recommended maximum pulse rate leads to greater benefits than having a pulse rate of 40 percent of the maximum?
B. Yes. The argument demonstrates a correlation between the mood of professional cyclists and how intense the cycling is. When the cycling is at 60 percent of the recommended maximum pulse rate for recreational bikers, the professional cyclists reported being less depressed and angry. Those cycling at 85 percent, on the other hand, reported feeling more depressed and angry.
C. No. This answer choice is too strong. You can’t prove anything about pulse rates higher than 85 percent so it is possible that something higher could also improve mood.
D. No. This answer choice is too strong. You can’t prove that physical factors contribute as much as psychological factors contribute. The passage does not include information about any psychological factors that may have been at play so you can’t make that comparison.
E. No. You can’t prove whether moderate cycling benefits professional cyclists physically as much or more than intense cycling.
LSAT Logical Reasoning Prep Test 52 Question #16 16. Anyone who believes in extraterrestrials believes in
UFOs. But the existence of UFOs has been
conclusively refuted. Therefore a belief in
extraterrestrials is false as well.
Which one of the following arguments contains
flawed reasoning most similar to that in the
argument above?
(A) Anyone who believes in unicorns believes in
centaurs. But it has been demonstrated that
there are no centaurs, so there are no
unicorns either.
(B) Anyone who believes in unicorns believes in
centaurs. But you do not believe in centaurs,
so you do not believe in unicorns either.
(C) Anyone who believes in unicorns believes in
centaurs. But you do not believe in unicorns,
so you do not believe in centaurs either.
(D) Anyone who believes in unicorns believes in
centaurs. But there is no good reason to
believe in centaurs, so a belief in unicorns is
unjustified as well.
(E) Anyone who believes in unicorns believes in
centaurs. But it has been conclusively proven
that there is no such thing as a unicorn, so a
belief in centaurs is mistaken as well.
Correct Answer: A
A Parallel Flaw
This argument is diagrammable: Believe in existence of ETs → believe in existence of UFOs. UFOs don’t exist → ETs don’t exist. The argument does not support the assumption that the existence of ETs depends on the existence of UFOs.
A. Yes. Believe in existence of unicorns → believe in existence of centaurs. Centaurs don’t exist → unicorns can’t exist. This is the same argument structure and the same flaw.
B. No. This argument is not flawed. Believe in unicorns → believe in centaurs. Don’t believe in centaurs → don’t believe in unicorns. The second conditional is the contrapositive of the first.
C. No. This answer choice does not make the switch from belief to actual existence.
D. No. This answer choice does not make the switch from belief to actual existence.
E. No. This answer choice does make the switch from belief to actual existence but its second premise starts with the non-existence of unicorns, which were, in fact, the sufficient condition in the first premise. This does not match the structure of the original argument.
LSAT Logical Reasoning Prep Test 52 Question #17 17. People want to be instantly and intuitively liked.
Those persons who are perceived as forming
opinions of others only after cautiously gathering
and weighing the evidence are generally resented.
Thus, it is imprudent to appear prudent.
Which one of the following, if assumed, enables the
argument’s conclusion to be properly drawn?
(A) People who act spontaneously are well liked.
(B) Imprudent people act instantly and intuitively.
(C) People resent those less prudent than
themselves.
(D) People who are intuitive know instantly when
they like someone.
(E) It is imprudent to cause people to resent you.
Correct Answer: E
E Suffiicient Assumption
The conclusion of the argument is that it is imprudent to appear prudent. The first premise is that people want to be instantly and intuitively liked. The second premise is that people who are perceived to form opinions about others non-spontaneously are generally resented. The argument shifts language from the premise, which discusses behavior that causes resentment, to the conclusion, which discusses prudence. You need an answer that proves the conclusion by tying it to resentment.
A. No. The conclusion isn’t about how people need to act in order to be well liked. It is a judgment about appearing prudent, not about spontaneity.
B. No. You need to know why it is imprudent to appear prudent, not how imprudent people generally act.
C. No. If anything, this argument seems to claim that people resent those more prudent than themselves.
D. No. The conclusion is concerned with prudence and imprudence, not about intuitive people.
E. Yes. This gives the argument the connection it needs between imprudence and resentment.
LSAT Logical Reasoning Prep Test 52 Question #18 18. Journalist: Recent studies have demonstrated that a
regular smoker who has just smoked a
cigarette will typically display significantly
better short-term memory skills than a
nonsmoker, whether or not the nonsmoker has
also just smoked a cigarette for the purposes of
the study. Moreover, the majority of those
smokers who exhibit this superiority in shortterm memory skills will do so for at least eight
hours after having last smoked.
If the journalist’s statements are true, then each of
the following could be true EXCEPT:
(A) The short-term memory skills exhibited by a
nonsmoker who has just smoked a cigarette
are usually substantially worse than the shortterm memory skills exhibited by a nonsmoker
who has not recently smoked a cigarette.
(B) The short-term memory skills exhibited by a
nonsmoker who has just smoked a cigarette
are typically superior to those exhibited by a
regular smoker who has just smoked a
cigarette.
(C) The short-term memory skills exhibited by a
nonsmoker who has just smoked a cigarette
are typically superior to those exhibited by a
regular smoker who has not smoked for more
than eight hours.
(D) A regular smoker who, immediately after
smoking a cigarette, exhibits short-term
memory skills no better than those typically
exhibited by a nonsmoker is nevertheless
likely to exhibit superior short-term memory
skills in the hours following a period of heavy
smoking.
(E) The short-term memory skills exhibited by a
regular smoker who last smoked a cigarette
five hours ago are typically superior to those
exhibited by a regular smoker who has just
smoked a cigarette.
Correct Answer: B
B Inference
Find the answer choice that contradicts evidence in the argument.
A. No. This answer choice could be true. The argument doesn’t compare the difference in memory between nonsmokers who have just smoked a cigarette and nonsmokers who haven’t recently smoked a cigarette.
B. Yes. The argument states that the short-term memory skills of a nonsmoker who has just smoked are typically significantly worse than those of a smoker who has just smoked. So, this answer choice directly contradicts the argument.
C. No. The argument doesn’t compare the memory skills of nonsmokers who have just smoked and smokers who haven’t smoked in over eight hours. This answer choice could be true.
D. No. This answer choice could be true. The argument doesn’t say anything about periods of heavy smoking. The answer choice is also comparing two individuals, not the typical results.
E. No. The argument doesn’t compare the memory skills of smokers who last smoked five hours ago and smokers who have just smoked. This answer choice could be true.
LSAT Logical Reasoning Prep Test 52 Question #19 19. Educator: It has been argued that our professional
organization should make decisions about
important issues—such as raising dues and
taking political stands—by a direct vote of all
members rather than by having members vote
for officers who in turn make the decisions.
This would not, however, be the right way to
decide these matters, for the vote of any given
individual is much more likely to determine
organizational policy by influencing the
election of an officer than by influencing the
result of a direct vote on a single issue.
Which one of the following principles would, if valid,
most help to justify the educator’s reasoning?
(A) No procedure for making organizational
decisions should allow one individual’s vote
to weigh more than that of another.
(B) Outcomes of organizational elections should
be evaluated according to their benefit to the
organization as a whole, not according to the
fairness of the methods by which they are
produced.
(C) Important issues facing organizations should
be decided by people who can devote their
full time to mastering the information
relevant to the issues.
(D) An officer of an organization should not make
a particular decision on an issue unless a
majority of the organization’s members
would approve of that decision.
(E) An organization’s procedures for making
organizational decisions should maximize the
power of each member of the org
Correct Answer: E
E Principle Strengthen
This argument is designed to disagree with a claim. The educator is against deciding matters in his professional organization by a direct vote instead of having matter decided by officers who are elected by direct vote. The premise is that organizational policy will be more influenced by individuals voting for officers rather than individuals directly voting on issues. A principle that would strengthen it would connect procedures for making organizational decisions with the amount of influence each member has on these decisions.
A. No. This principle would weaken the argument, if anything.
B. No. This principle does not connect the procedures for making decisions with the amount of influence of each member on these decisions.
C. No. This answer choice does not tell us that it would be the officers that would have this time, so it doesn’t help the argument.
D. No. This might be true but it does not strengthen the educator’s claim that voting to elect officers will give each individual more influence in organizational policy.
E. Yes. This principle strengthens the argument by relating procedures for making decisions and the maximization of the power of each individual to influence the decisions.
LSAT Logical Reasoning Prep Test 52 Question #20 20. Neural connections carrying signals from the cortex
(the brain region responsible for thought) down to
the amygdala (a brain region crucial for emotions)
are less well developed than connections carrying
signals from the amygdala up to the cortex. Thus, the
amygdala exerts a greater influence on the cortex
than vice versa.
The argument’s conclusion follows logically if which
one of the following is assumed?
(A) The influence that the amygdala exerts on the
rest of the brain is dependent on the influence
that the cortex exerts on the rest of the brain.
(B) No other brain region exerts more influence
on the cortex than does the amygdala.
(C) The region of the brain that has the most
influence on the cortex is the one that has the
most highly developed neural connections to
the cortex.
(D) The amygdala is not itself controlled by one or
more other regions of the brain.
(E) The degree of development of a set of neural
connections is directly proportional to the
influence transmitted across those
connections.
Correct Answer: E
E Sufficient Assumption
The conclusion of the argument is that the amygdala exerts a greater influence on the cortex than vice versa. The premise for this conclusion is that the neural connections that carry signals from the cortex to the amygdala are less well developed than the connections carrying signals the other way around. The argument equates how developed the connections carrying signals from one part of the brain to the other are to the amount of influence one part of the brain has on the other.
A. No. The argument is concerned only with the influence the amygdala exerts on the cortex, not the rest of the brain.
B. No. Other brain regions are not relevant and the assumption needs to equate how developed the connections are to the amount of influence.
C. No. This answer choice is too general. The region of the brain that has the most highly developed neural connections to the cortex might be something other than the amygdala.
D. No. The argument is concerned with the influence that the amygdala has on the cortex. It doesn’t matter whether some other region controls it.
E. Yes. This connects the degree of development of neural connections with the degree of influence one part of the brain has on another.
LSAT Logical Reasoning Prep Test 52 Question #21 21. The Iliad and the Odyssey were both attributed to
Homer in ancient times. But these two poems differ
greatly in tone and vocabulary and in certain details
of the fictional world they depict. So they are almost
certainly not the work of the same poet.
Which one of the following statements, if true, most
weakens the reasoning above?
(A) Several hymns that were also attributed to
Homer in ancient times differ more from the
Iliad in the respects mentioned than does the
Odyssey.
(B) Both the Iliad and the Odyssey have come
down to us in manuscripts that have suffered
from minor copying errors and other textual
corruptions.
(C) Works known to have been written by the
same modern writer are as different from
each other in the respects mentioned as are
the Iliad and the Odyssey.
(D) Neither the Iliad nor the Odyssey taken by
itself is completely consistent in all of the
respects mentioned.
(E) Both the Iliad and the Odyssey were the result
of an extended process of oral composition in
which many poets were involved.
Correct Answer: C
C Weaken
This argument disagrees with the claim that the difference in vocabulary, tone, and details of the fictional world depicted in the Iliad and the Odyssey imply that they could almost certainly not be the work of the same poet. A good answer choice will demonstrate how even if two works are different in many ways, the same person may have written both of them.
A. No. Homer might not have actually written the hymns either.
B. No. This doesn’t go far enough. If the manuscripts have suffered only minor copying errors and other textual corruptions, the corruptions won’t explain away all of the stylistic differences between the two.
C. Yes. You know that the modern writer actually wrote the works described. This counterexample lends plausibility to the possibility that Homer wrote both the Iliad and the Odyssey despite their many differences.
D. No. The argument rests on comparing the Iliad with the Odyssey, not looking at each by itself.
E. No. This would support the claim that Homer didn’t write both.
LSAT Logical Reasoning Prep Test 52 Question #22 22. Moralist: A statement is wholly truthful only if it is
true and made without intended deception. A
statement is a lie if it is intended to deceive or
if its speaker, upon learning that the statement
was misinterpreted, refrains from clarifying it.
Which one of the following judgments most closely
conforms to the principles stated by the moralist?
(A) Ted’s statement to the investigator that he had
been abducted by extraterrestrial beings was
wholly truthful even though no one has ever
been abducted by extraterrestrial beings.
After all, Ted was not trying to deceive the
investigator.
(B) Tony was not lying when he told his
granddaughter that he did not wear dentures,
for even though Tony meant to deceive his
granddaughter, she made it clear to Tony that
she did not believe him.
(C) Siobhan did not tell a lie when she told her
supervisor that she was ill and hence would
not be able to come to work for an important
presentation. However, even though her
statement was true, it was not wholly truthful.
(D) Walter’s claim to a potential employer that he
had done volunteer work was a lie. Even
though Walter had worked without pay in his
father’s factory, he used the phrase “volunteer
work” in an attempt to deceive the interviewer
into thinking he had worked for a socially
beneficial cause.
(E) The tour guide intended to deceive the tourists
when he told them that the cabin they were
looking at was centuries old. Still, his
statement about the cabin’s age was not a lie,
for if he thought that this statement had been
misinterpreted, he would have tried to
clarify it.
Correct Answer: D
D Principle Match
Diagram the moralist’s two principles. 1. Statement wholly truthful → it’s true and made without intended deception. Contrapositive: ~true or made with intended deception → ~not wholly truthful. 2. Intended to deceive or doesn’t clarify misinterpretation → lie. Contrapositive: ~lie → ~intended to deceive and clarifies misinterpretation. The best answer choice will provide an example in which at least one of the two principles is fulfilled.
A. No. Neither principle gives criteria to determine that a statement is wholly truthful.
B. No. Neither principle gives criteria to determine that a statement is not a lie, only when it is a lie.
C. No. You don’t know whether Siobhan intended to deceive, nor do you know whether she is actually sick. This doesn’t fit either principle.
D. Yes. If a statement is intended to deceive, it is a lie. Walter intended to deceive, so he lied.
E. No. If the statement is intended to deceive OR the person doesn’t clarify a misinterpretation, the statement is a lie. So the tour guide DID lie, according to the second principle.
LSAT Logical Reasoning Prep Test 52 Question #23 23. Principle: It is healthy for children to engage in an
activity that promotes their intellectual
development only if engaging in that activity
does not detract from their social
development.
Application: Although Megan’s frequent reading
stimulates her intellectually, it reduces the
amount of time she spends interacting with
other people. Therefore, it is not healthy for
her to read as much as she does.
The application of the principle is most vulnerable to
criticism on which one of the following grounds?
(A) It misinterprets the principle as a universal
claim intended to hold in all cases without
exception, rather than as a mere
generalization.
(B) It overlooks the possibility that the benefits of
a given activity may sometimes be important
enough to outweigh the adverse health effects.
(C) It misinterprets the principle to be, at least in
part, a claim about what is unhealthy, rather
than solely a claim about what is healthy.
(D) It takes for granted that any decrease in the
amount of time a child spends interacting
with others detracts from that child’s social
development.
(E) It takes a necessary condition for an activity’s
being healthy as a sufficient condition for its
being so.
Correct Answer: D
D Principle Match
This argument contains a principle that states the following: healthy to engage in intellectual development → engaging in that activity does not detract from social development. The argument then presents an application in the evidence about Megan. It draws the conclusion that Megan’s amount of reading is not healthy because it reduces the amount of time she spends interacting with other people. The flaw is that the argument equates interacting with others with social development.
A. No. The principle is a universal claim.
B. No. The argument does not discuss health effects.
C. No. As the principle is a conditional statement, it has a contrapositive. The contrapositive does make a claim about what is unhealthy.
D. Yes. The argument equates interacting with other people and social development. It might well be that the plot lines in the books that she reads help with her social development.
E. No. This argument does not contain a necessary/sufficient flaw.
LSAT Logical Reasoning Prep Test 52 Question #24 In response to several bacterial infections traced to
its apple juice, McElligott now flash pasteurizes its
apple juice by quickly heating and immediately
rechilling it. Intensive pasteurization, in which juice
is heated for an hour, eliminates bacteria more
effectively than does any other method, but is likely
to destroy the original flavor. However, because
McElligott’s citrus juices have not been linked to any
bacterial infections, they remain unpasteurized.
The statements above, if true, provide the most
support for which one of the following claims?
(A) McElligott’s citrus juices contain fewer
infectious bacteria than do citrus juices
produced by other companies.
(B) McElligott’s apple juice is less likely to contain
infectious bacteria than are McElligott’s citrus
juices.
(C) McElligott’s citrus juices retain more of the
juices’ original flavor than do any pasteurized
citrus juices.
(D) The most effective method for eliminating
bacteria from juice is also the method most
likely to destroy flavor.
(E) Apple juice that undergoes intensive
pasteurization is less likely than McElligott’s
apple juice is to contain bacteria.
Correct Answer: E
E Inference
Find the answer choice supported by the passage.
A. No. The passage does not provide information about the amount of bacteria in other companies’ juices.
B. No. The passage doesn’t have any comparison between the amounts of bacteria in the apple juice versus the citrus juices. You can’t prove which juice is less likely to contain infectious bacteria.
C. No. Intensive pasteurization is likely to destroy the original flavor, but whether other types of pasteurization do this is not discussed in the passage. So there is not enough information to determine whether McElligott’s unpasteurized citrus juices retain more of the original flavor than do any pasteurized citrus juices.
D. No. This answer choice sounds good but is too strong. Intensive pasteurization is the most effective method for eliminating bacteria from juice. Intensive pasteurization is also likely to destroy the original flavor of the juice. The passage does not support, however, that intensive pasteurization is the method most likely to destroy flavor. Another method of eliminating bacteria might be even more likely to destroy the flavor.
E. Yes. McElligott’s juice did not undergo intensive pasteurization because it was flash pasteurized. It is also stated that intensive pasteurization is the most effective way to eliminate bacteria from the juice, so a juice that undergoes intensive pasteurization is less likely to contain bacteria than McElligott’s apple juice.